Why A and not B?
I got all other questions on this passage correct, and I understood them, but I am wondering why ...
Alec on August 16 at 10:38AM
  • June 1997 LSAT
  • SEC1
  • Q17
2
Replies
Could someone please explain this?
Thanks
jingjingxiao11111@gmail.com on March 30, 2022
  • June 1997 LSAT
  • SEC1
  • Q9
2
Replies
Could someone please explain this?
Thanks
jingjingxiao11111@gmail.com on February 14, 2022
  • June 1997 LSAT
  • SEC1
  • Q15
1
Reply
Could someone please explain this? Thanks
Could someone please explain this? Thanks
jingjingxiao11111@gmail.com on January 21, 2022
  • June 1997 LSAT
  • SEC1
  • Q20
1
Reply
Can someone please explain this question
Thanks!
liwenong28 on January 7, 2021
  • June 1997 LSAT
  • SEC1
  • Q23
1
Reply
Could someone please explain this?
Could someone please explain this? Thanks
jingjingxiao11111@gmail.com on December 10, 2020
  • June 1997 LSAT
  • SEC1
  • Q24
1
Reply
June 1997 SEC 1 Q8
Why can't A be the correct answer? Also, how can we know that the author is advocating a certain ...
kens on July 24, 2020
  • June 1997 LSAT
  • SEC1
  • Q8
1
Reply
June 1997 LSAT rc7
Can you explain this question in more detail? Thanks in advance!
kens on July 24, 2020
  • June 1997 LSAT
  • SEC1
  • Q7
1
Reply
Why not "A"
The article specifically states, in line 30, that "The economists' view is that even if such a CE...
DDL on September 29, 2019
  • June 1997 LSAT
  • SEC1
  • Q19
1
Reply
Help
I do understand the first part of the 3rd paragraph is to elaborate the position of linguists who...
Batman on November 9, 2017
  • June 1997 LSAT
  • SEC1
  • Q25
2
Replies
Help
I could hardly distinguish the difference between (d) and (e). Why is (d) the answer to this ques...
Batman on August 15, 2014
  • June 1997 LSAT
  • SEC1
  • Q13
2
Replies
Help
Could you please explain the reason why (E) shouldn't be the answer? Thanks,
Batman on August 14, 2014
  • June 1997 LSAT
  • SEC1
  • Q8
2
Replies
Help
How could both approach be complement each other according to the passage? I don't understand tha...
Batman on August 14, 2014
  • June 1997 LSAT
  • SEC1
  • Q2
2
Replies